Local Anesthetics Flashcards

1
Q

A 50-year-old woman is scheduled to undergo bilateral deep inferior epigastric artery perforator (DIEP) flap breast reconstruction. A donor site regional block with liposomal bupivacaine is planned for analgesia. Which of the following injection locations is most likely to improve pain control and decrease narcotic use in this patient?
A) Between external oblique and internal oblique fascias
B) Between internal oblique and transversus abdominis fascias
C) Deep to rectus abdominis fascia
D) Deep to transversus abdominis fascia
E) Subcutaneous plane

A

The correct response is Option B.

The patient is undergoing a transversus abdominis plane (TAP) block. TAP blocks have become popular adjunctive pain control measures for a variety of abdominal surgeries. They can either be performed transcutaneously with ultrasound guidance or under direct visualization in the operating room prior to closure of the abdominal donor site in deep inferior epigastric artery perforator (DIEP) or transverse rectus abdominis musculocutaneous (TRAM) flap surgeries. The segmental innervation to be blocked in the TAP block is located between the internal oblique and transversus abdominis muscles. Either infiltration of liposomal bupivacaine or placement of epidural catheters in the TAP space has been described.

How well did you know this?
1
Not at all
2
3
4
5
Perfectly
2
Q

A healthy 154-lb (70-kg), 30-year-old woman is undergoing liposuction of multiple sites. Five minutes after the local anesthetic infusion, she sustains a cardiac arrest. Local anesthetic systemic toxicity is diagnosed. After CPR is initiated, administration of which of the following is the most appropriate next step in management?

A

The correct response is Option B.

This patient has sustained a cardiovascular collapse secondary to local anesthetic systemic toxicity (LAST). The maximum safe dose of lidocaine with epinephrine in this patient is 35 to 55 mg/kg.

Lipid Rescue protocol recommends an initial intralipid 20% bolus of 1.5 mL/kg over 1 minute. This should be followed immediately with a continuous infusion at 0.25 mL/kg/min. A single bolus is typical, but should be repeated or the infusion increased if spontaneous circulation fails to return or blood pressure declines.

Cardiovascular collapse from LAST differs from that secondary to myocardial ischemia. In LAST, raising the peripheral vascular resistance with vasopressors like vasopressin, can impair cardiac output and impede resuscitation. Epinephrine should be given in small doses (<1 mcg/kg). Pharmacologic agents that reduce contractility (beta blockers {propranolol}, calcium channel blockers {verapamil}, or propofol) should be avoided when there is evidence of cardiovascular instability.

Adenosine is not useful in the management of LAST patients. It is typically used to treat supraventricular tachycardia.

How well did you know this?
1
Not at all
2
3
4
5
Perfectly
3
Q

A 55-year-old woman is evaluated for an incisional hernia that developed after a complicated right hemicolectomy 5 years ago. Medical history includes hypertension. BMI is 34 kg/m2. She reports shortness of breath when trying to scrub floors at work but is able to climb a flight of stairs or walk up a hill without symptoms. There are no other respiratory or cardiovascular issues. Cardiovascular examination shows no abnormalities. Recent electrocardiography, complete blood count, and basic metabolic panel show no abnormalities. Hernia repair with separation of components is being considered. Which of the following is the most appropriate next step in the preoperative cardiac assessment of this patient?
A) Cardiac catheterization
B) Echocardiographic exercise stress testing
C) Echocardiography at rest
D) Nuclear cardiac pharmacologic stress testing
E) No further cardiac testing is needed

A

The correct response is Option E.

No further preoperative cardiac testing is needed for this patient being considered for an elective, greater-than-low-risk procedure, without any evidence of active cardiac conditions or clinical risk factors, and with moderate functional capacity (metabolic equivalents, or METs, >= 4).

Assessment of left ventricular function at rest, whether by echocardiogram, radionuclide angiogram, or contrast ventriculography, has not been shown to be a consistent predictor of perioperative ischemic events. Cardiac stress testing, including by exercise or drug induced, is not indicated in this patient.

In 2007, the American College of Cardiology and the American Heart Association published updated guidelines on perioperative cardiovascular evaluation and care for noncardiac surgery (NCS), suggesting a stepwise algorithmic approach to perioperative cardiac risk assessment:

Is there need for emergency NCS? If yes, proceed with surgery.
Are there active cardiac conditions (unstable coronary syndromes, decompensated heart failure, significant arrhythmias, severe valvular disease)? If yes, evaluate and treat per specific guidelines before considering NCS.
Is the NCS considered low risk (e.g., superficial and ophthalmologic procedures)? If yes, proceed with low-risk NCS.
Does the patient have good functional capacity (METs >= 4) without symptoms? If yes, proceed with NCS.
Is there need for emergency NCS? If yes, proceed with surgery.
Three or more clinical risk factors undergoing vascular surgery: consider testing if it will change management
One or two clinical risk factors undergoing vascular surgery or one or more clinical risk factors undergoing intermediate-risk surgery: either proceed with surgery with heart rate control (beta blockade) or consider noninvasive testing if it will change management
Preoperative cardiac assessment should include an estimation of the patient’s functional capacity, based on his/her ability to perform a spectrum of common daily tasks. This has been shown to correlate well with maximum oxygen uptake by treadmill testing. Four METs is a common threshold used in many decision-making points in the perioperative cardiac evaluation.

How well did you know this?
1
Not at all
2
3
4
5
Perfectly
4
Q

A 65-year-old woman is scheduled to undergo unilateral breast reconstruction with tissue expander. Medical history includes placement of a drug-eluting stent 4 months ago. Current medications include aspirin and clopidogrel. The surgical oncologist does not want to wait longer to perform the mastectomy. Which of the following is the most appropriate management of preoperative medications in this patient?
A) Continue both aspirin and clopidogrel
B) Discontinue aspirin and continue clopidogrel
C) Discontinue both aspirin and clopidogrel
D) Discontinue both aspirin and clopidogrel; start prophylactic enoxaparin
E) Discontinue clopidogrel and continue aspirin

A

The correct response is Option A.

A patient who underwent placement of a drug-eluting stent 4 months ago should continue on aspirin and clopidogrel for at least 6 months, except in cases with high risk of bleeding (intracranial or spine surgery). The combination of aspirin and clopidogrel appears to reduce the rates of cardiovascular ischemic events. Patients with coronary stents have an 8 to 10% risk of developing major adverse cardiovascular events and stent thrombosis after elective noncardiac surgery, which exceeds the 1 to 5% risk of major adverse cardiovascular events in non-stented patients having noncardiac surgery.

How well did you know this?
1
Not at all
2
3
4
5
Perfectly
5
Q

A critically ill 69-year-old man is observed to have new-onset atrial fibrillation complicated by rapid ventricular response. Angina and hemodynamic instability with hypotension and acute ST depression is noted. Which of the following is the next step in management?
A) Emergency cardiac catherization with AV nodal ablation followed by permanent ventricular pacing
B) Heparin anticoagulation and emergency transesophageal echocardiography (TEE) to rule out left atrial thrombus
C) Immediate external electrical cardioversion
D) Intravenous administration of amiodarone with loading dose and titrated infusion until sinus rhythm is restored
E) Intravenous administration of a beta blocker or nondihydropyridine calcium channel blocker to slow the ventricular heart rate

A

The correct response is Option C.

Immediate cardioversion is recommended for patients with atrial fibrillation (AF) or atrial flutter with rapid ventricular response associated with hemodynamic instability such as hypotension, ongoing myocardial ischemia, or decompensated heart failure. Both chemical and electrical cardioversion can be attempted according to established protocols until sinus rhythm is restored with a goal of optimizing hemodynamic parameters. If initial cardioversion is unsuccessful, repeated attempts at cardioversion may be made after adjusting the location of the electrodes, applying pressure over the electrodes, or following administration of an antiarrhythmic medication according to Advanced Cardiac Life Support (ACLS) algorithms.

A trial of medical management for rate control is appropriate in the absence of hemodynamic instability. In general, beta blockers are the most common agents used for rate control, followed by nondihydropyridine calcium channel blockers such as diltiazem, as well as other agents such as digoxin and amiodarone. Patient comorbidities influence the choice of medical therapy to minimize risk of adverse events such as heart failure decompensation, exacerbation of chronic obstructive pulmonary disease, or acceleration of conduction in patients with preexcitation physiology.

Appropriate anticoagulation management around the time of elective cardioversion is essential for reducing thromboembolic risk. When AF has clearly existed at < 48 hours’ duration, it is common practice to perform cardioversion without transesophageal echocardiography (TEE) or antecedent anticoagulation. When AF has existed at >48 hours’ duration or duration is unknown, then anticoagulation for ? 3 weeks before and continuing for ? 4 weeks after cardioversion is recommended unless otherwise contraindicated. In case of relative contraindications to anticoagulation, TEE can be performed to help guide relative risk assessment related to thromboembolism of preexisting cardiac thrombi arising in the setting of prolonged atrial dysfunction. Importantly, when emergent cardioversion is indicated because of hemodynamic instability, initiation of anticoagulation and TEE should not delay acute interventions to stabilize the patient’s condition.

AV nodal ablation with permanent ventricular pacing can be considered to improve rate control when other attempts at maintaining rate control through medical management are unsuccessful. This intervention is usually reserved for elderly patients, because it leads to pacemaker dependency. However, patients with symptoms refractory to medical therapy who are treated with AV nodal ablation and permanent pacemaker implantation have demonstrated an improvement in cardiac symptoms, quality of life, and health care utilization.

How well did you know this?
1
Not at all
2
3
4
5
Perfectly
6
Q
A 42-year-old woman is scheduled to undergo reduction mammaplasty. Medical history includes macromastia and von Willebrand disease. Preoperatively, administration of desmopressin (DDAVP) is planned for bleeding prophylaxis. The total preoperative dose is best determined by measuring the activity levels of vWF:RCoF (von Willebrand factor ristocetin cofactor) and which of the following coagulation factors?
A) II
B) V
C) VII
D) VIII
E) X
A

The correct response is Option D.

In surgical patients with von Willebrand disease (vWD) receiving desmopressin (DDAVP) for bleeding prophylaxis, total preoperative dose should be determined by preoperative measurement of activity levels of von Willebrand factor ristocetin cofactor (vWF:RCoF) and coagulation factor VIII.

The other listed coagulation factors are not associated with isolated vWD.

vWD is the most common congenital bleeding disorder, with estimated prevalence as high as 1.3%. It is caused by deficiency (types I and III) or dysfunction (type II) of von Willebrand factor (vWF), leading to impaired platelet adhesion and possibly lower levels of coagulation factor VIII. In its normal state, vWF is a plasma protein that mediates the initial adhesion of platelets at sites of vascular injury and also binds and stabilizes coagulation factor VIII in the circulation. Both vascular endothelial cells and platelets synthesize and store vWF.

DDAVP is a synthetic derivative of ADH (vasopressin, antidiuretic hormone) that is used for treatment and prophylaxis of bleeding in patients with vWD. It promotes release of stored vWF from endothelial cells into the plasma, being particularly effective in patients with partial quantitative deficiency of vWF, or type I (about 75% of cases of vWD). Patients who do not appropriately respond to DDAVP administration should receive vWF concentrate.

For bleeding prophylaxis for minor surgery, the preoperative goal should be to achieve vFW:RCoF and factor VIII activity levels of at least 30 IU/dL (preferably >50 IU/dL). Such levels should be maintained for 1 to 5 days postoperatively. For prophylaxis for major surgery, preoperative activity levels should be at least 100 IU/dL and maintained above 50 IU/dL for at least 7 to 10 days.

To decrease risk of perioperative thromboembolism, vWF:RCoF levels should not exceed 200 IU/dL and factor VIII activity should not exceed 250 IU/dL. Fluid restriction is advised for patients receiving DDAVP perioperatively, to avoid hyponatremia and seizures.

How well did you know this?
1
Not at all
2
3
4
5
Perfectly
7
Q

A 55-year-old woman is evaluated for mild cellulitis after undergoing cosmetic excision of a facial mole 7 days ago. Cephalexin is prescribed. Approximately 30 minutes after taking her first dose, she returns to the office because of sudden and progressive onset of generalized hives, periorbital edema, and flushing. The patient appears confused and then collapses. Blood pressure is 85/50 mmHg. An audible wheeze is noted. Which of the following is the most appropriate next step in management?
A) Administration of albuterol nebulizer
B) Administration of diphenhydramine and ranitidine intravenously
C) Injection of epinephrine (1 mg/mL) 0.3 to 0.5 mg intramuscularly
D) Injection of methylprednisolone 125 mg intramuscularly
E) Rapid infusion of 0.9% saline 1 to 2 L intravenously

A

The correct response is Option C.

This patient is presenting with anaphylaxis. The first and most crucial step in managing acute anaphylaxis is the administration of epinephrine.

Anaphylaxis is a serious and life-threatening hypersensitivity reaction. Medications and insect stings are the most common triggers in adults, with beta-lactam antibiotic exposure among the most implicated medications. In this setting, anaphylaxis is diagnosed when two or more of the following are observed minutes to hours after exposure:

Skin-mucosal tissue reactions (such as generalized urticaria, swollen lip, pruritus, and flushing)
Respiratory compromise (such as dyspnea, wheeze, bronchospasm, stridor, and hypoxemia)
Hypotension or associated findings (such as syncope, confusion, collapse, and incontinence)
Persistent gastrointestinal reactions (such as crampy abdominal pain and vomiting)
Intramuscular epinephrine can be repeated every 5 to 15 minutes as needed. If given promptly, most patients respond to one or two doses. Simultaneous with this treatment, an emergency team can be summoned. The patient should be placed in a recumbent position with legs elevated and supplemental oxygen administered.

Volume resuscitation can be initiated to compensate for severe loss of intravascular volume that can accompany fluid shifts of untreated anaphylaxis and to support blood pressure not responsive to epinephrine.

Bronchodilators, H1 and H2 antihistamines such as diphenhydramine and ranitidine, and glucocorticoids such as methylprednisolone are all reasonable but second-line treatments for acute anaphylaxis.

Intubation should be performed immediately in the setting of progressive upper airway closure such as stridor and tongue edema, or if respiratory arrest is present. However, first-dose epinephrine administration should not be delayed for intubation. Epinephrine works rapidly to reduce airway edema that might otherwise prevent successful intubation, if not completely reverse the underlying etiology of cardiopulmonary collapse in anaphylaxis.

While cardiovascular disease is a risk factor for poor outcome from anaphylaxis, it is not a contraindication for epinephrine administration. It is generally accepted that the risk of death or brain damage from prolonged or undertreated anaphylaxis outweighs the risk of appropriately administered epinephrine. If there is inadequate response to initial intramuscular epinephrine and volume resuscitation, then intravenous epinephrine can be introduced in an intensive care setting by slow infusion at 2 to 10 mcg per minute and titrated to effect.

How well did you know this?
1
Not at all
2
3
4
5
Perfectly
8
Q
A critically ill 65-year-old woman is brought to the intensive care unit, where she sustains respiratory arrest. Temperature is 101.3ºF (38.5ºC), heart rate is 105 bpm, and blood pressure is 85/60 mmHg. Hematocrit is 35%. She is immediately intubated. Noninvasive pulse oximetry is initiated to monitor oxygen saturation (SaO2). Which of the following conditions is most likely to alter pulse oximetry values in this patient?
A) Anemia
B) Fever
C) Hypotension
D) Tachycardia
E) Tachypnea
A

The correct response is Option .

Hypotension is most likely to alter pulse oximetry values by reducing peripheral arterial blood flow.

Pulse oximetry measures the relative transmission of light at two wavelengths that differ significantly when passed through loaded versus non-loaded hemoglobin (e.g., oxyhemoglobin versus deoxyhemoglobin). In order to provide values that correlate with arterial oxygen saturation levels as opposed to tissue bed or venous saturation levels, standard pulse oximeters require pulsatile blood to distinguish transmission at the peak of arterial pulsation relative to baseline transmission levels. Thus, pulse oximetry measurements will change both with changes in hemoglobin oxygen saturation and with conditions that interfere with the device’s ability to detect pulsatile blood flow.

Other conditions that alter pulse oximetry measurements by reducing the detection of fluctuations from arterial blood flow include peripheral vasoconstriction from hypothermia and vasopressor and interference from motion, such as tremors or shivering. Incorrect sensor application, highly calloused skin, and nail polish can also affect measurements by interfering with transmission readings.

Because standard pulse oximetry only measures the relative difference in transmission between oxygenated and deoxygenated hemoglobin and not the absolute value of oxygenated hemoglobin, anemia does not significantly affect pulse oximetry values within physiologic ranges.

Tachycardia, tachypnea, and fever do not directly affect pulse oximetry values.

How well did you know this?
1
Not at all
2
3
4
5
Perfectly
9
Q
A 63-year-old man with end-stage renal failure is evaluated for unilateral leg swelling and shortness of breath 2 days after undergoing bilateral axillary hidradenitis excision. Current medication includes aspirin. Oxygen saturation on room air is 88%. During diagnostic evaluation, empiric administration of which of the following drugs is most appropriate?
A) Intravenous heparin
B) Oral clopidogrel
C) Oral rivaroxaban
D) Oral warfarin
E) Subcutaneous enoxaparin
A

The correct response is Option A.

The patient has a presumed diagnosis of deep vein thrombosis (DVT) and pulmonary embolism (PE). Enoxaparin and rivaroxaban are contraindicated in patients with renal failure. Warfarin is used for long-term treatment of established DVT. Clopidogrel is an antiplatelet inhibitor that inhibits blood clots in coronary artery disease, peripheral vascular disease, cerebrovascular disease, and prevents myocardial infarction. Heparin is safe in renal failure patients and is indicated for treatment in acute DVT/PE.

How well did you know this?
1
Not at all
2
3
4
5
Perfectly
10
Q

A 69-year-old man is evaluated in the intensive care unit 5 days after undergoing abdominal wall reconstruction for a multiple-recurrence ventral hernia. Medical history shows no cardiac disease. Temperature is 103°F (39°C), heart rate is 110 bpm, and mean arterial pressure (MAP) is 50 mmHg. Airway and ventilation are secured, and supplemental oxygen is initiated. Hemoglobin concentration is 9 g/dL. Which of the following is the most appropriate next step in management?
A) Initiation of inotropic support
B) Initiation of vasopressor support
C) Introduction of a pulmonary artery catheter to monitor wedge pressures
D) Volume resuscitation with crystalloid
E) Volume resuscitation with packed red blood cells

A

The correct response is Option D.

In this patient with septic shock, once airway and breathing are secured, the next most appropriate step in management is to restore effective circulation and perfusion to peripheral tissues starting with a trial of volume resuscitation with crystalloid.

Signs of impaired end organ perfusion in septic shock include hypotension (e.g., mean arterial pressure <70 mmHg), tachycardia (e.g., heart rate >100 bpm), warm flushed skin giving way to cool clammy skin as blood flow is redirected to core organs, obtundation, and an elevated serum lactate concentration (e.g., >1 mmol/L).

First-line therapy for restoration of tissue perfusion is volume resuscitation using intravenous crystalloid targeted to physiologic end points while monitoring for clinical or radiographic evidence of either cardiogenic or non-cardiogenic pulmonary edema (ie, ARDS).

The addition of pulmonary artery catheters has not been shown to improve outcomes in the routine management of septic shock and is associated with increased complications.

As in the given scenario, evidence and expert opinion do not support the transfusion of blood products greater than a hemoglobin concentration of 7 g/dL in the absence of concurrent hemorrhagic shock, cardiac history, or active myocardial ischemia. For example, a recent multicenter randomized study involving 998 patients with septic shock reported no significant difference in mortality or rate of ischemic events between patients transfused when hemoglobin concentration was <7 g/dL compared with patients transfused when hemoglobin was <9 g/dL. Their former (more restrictive) transfusion trigger resulted in 50% fewer red blood cell transfusions compared with the more liberal strategy.

Vasopressors (e.g., norepinephrine) are second-line agents in the treatment of septic shock refractory to trial of volume resuscitation as long as intravenous fluids successfully improve perfusion without impairing gas exchange.

Inotropic agents (e.g., dobutamine) are also second-line agents to initial volume resuscitation in this scenario. They may be useful with refractory shock in the setting of diminished cardiac output.

How well did you know this?
1
Not at all
2
3
4
5
Perfectly
11
Q
A 5-year-old boy is brought to the emergency department 45 minutes after accidentally injecting his palm with epinephrine from an auto-injector (EpiPen). On physical examination, the ring finger is soft and pale, and capillary refill time is poor. A small puncture mark is noted on the flexor surface of the palm just proximal to the metacarpophalangeal (MCP) joint. Which of the following is the most appropriate next step in management?
A) Application of a hot pack
B) Emergent operative exploration
C) Subcutaneous injection of nifedipine
D) Topical nitroglycerin paste
E) Observation
A

The correct response is Option E.

Accidental self-injection of epinephrine with an EpiPen occurs in 1 in 50,000 syringes. There have been no documented cases of digital necrosis following injection, and observation is indicated in this case. The effect of epinephrine’s vasoconstriction lasts for approximately 90 minutes and will likely resolve on its own.

Phentolamine has been described as a reversal agent for epinephrine and has shown clinical efficacy, but there has been no study to show that there are better outcomes with phentolamine injection compared with observation. Also, the added volume of injection with phentolamine could cause pressure necrosis and the timing between EpiPen injection and definitive treatment would usually be outside of the 90 minutes it would take for the epinephrine to wear off on its own.

Topical nitroglycerin paste and subcutaneous injection of calcium channel blockers such as nifedipine have not shown to be viable treatment modalities. Application of a hot pack can lead to increased tissue damage and burns and is not indicated. Emergent exploration is indicated for compartment syndrome and is not indicated in this case where the finger is soft.

How well did you know this?
1
Not at all
2
3
4
5
Perfectly
12
Q

A 35-year-old man undergoes a 90-minute rhytidectomy procedure with intravenous sedation. The patient smokes 10 cigarettes daily, but is otherwise healthy. Which of the following factors most likely places this patient at increased risk for postoperative nausea and vomiting?
A) Duration of procedure
B) Gender
C) History of cigarette smoking
D) History of postoperative nausea and vomiting
E) Type of anesthesia

A

The correct response is Option D.

Risk factors for postoperative nausea and vomiting include: female gender, nonsmoking status, prior history of postoperative nausea/vomiting/motion sickness, use of volatile anesthetics/general anesthesia, opioid/narcotic use, facial rejuvenation procedures, and long duration of surgery.

Several measures can be taken to decrease postoperative nausea and vomiting. A thorough history with identification of risk factors can aid stratification of patients preoperatively. Use of long-acting local anesthetic agents, nonsteroidal anti-inflammatory drugs (NSAIDs), and cyclooxygenase-2 selective inhibitors can decrease the need for postoperative opioid use. Avoidance of nitrous oxide, especially in combination with fentanyl and volatile inhalational gases, and multimodal use of serotonin antagonists combined with other antiemetic agents can also decrease postoperative nausea and vomiting.

How well did you know this?
1
Not at all
2
3
4
5
Perfectly
13
Q
A 37-year-old man, who is American Society of Anesthesiologists (ASA) Class 2, comes to the office for evaluation and treatment of human immunodeficiency virus (HIV)–associated lipodystrophy. The patient has a CD4 count of 100 cells/mm3. Autologous fat grafting is planned. Which of the following factors is most likely to increase this patient’s risk of postoperative complications?
A) ASA Class
B) CD4 cell count
C) HIV seropositivity
D) Percutaneous surgery
A

The correct response is Option B.

Higher American Society of Anesthesiologists (ASA) class has been identified as a risk factor for postoperative complications in HIV-positive patients in multiple studies. A patient who is ASA Class 2 has only mild systemic disease. Increasing class number indicates increasing severity of disease (Class 3 – severe systemic disease, Class 4 – severe systemic disease that is a constant threat to life).

Acquired immunodeficiency syndrome (AIDS) is diagnosed when the CD4 count is <200 cells/mm3 or with acquisition of an AIDS-defining illness. An absolute CD4 count of <200 cells/mm3 has been associated with increased risk of complications including wound infections.

HIV seropositivity alone has been found not to be an independent risk factor for postoperative complications.

Percutaneous surgery, such as fat grafting, has not been associated with increased risk of infection in HIV patients, nor has skin incisional surgery. Transoral mucosal incisional surgery has been found to be associated with a significantly greater risk of wound infection in HIV patients.

A viral load greater than 10,000 copies/mL suggests that antiretroviral therapy is no longer effective and has been identified as an independent risk factor for complications.

How well did you know this?
1
Not at all
2
3
4
5
Perfectly
14
Q
A 35-year-old woman comes to the office for consultation on augmentation mammaplasty. During preoperative workup, she reports that her mother has a history of malignant hyperthermia. The patient has never undergone surgery. Which of the following anesthetic agents is most appropriate for this surgery?
A) Desflurane
B) Halothane
C) Isoflurane
D) Propofol
E) Succinylcholine
A

The correct response is Option D.

Propofol can be safely used in patients with a suspected diagnosis of malignant hyperthermia.

Malignant hyperthermia is a rare, life-threatening inherited skeletal muscle disorder that shows symptoms of hypermetabolic reaction to volatile anesthetic gases and depolarizing muscle relaxants. The incidence is between 1 in 5000 to 1 in 100,000 anesthetic encounters. Mortality rates have decreased from 70% to less than 5% as awareness of this condition has led to accurate diagnosis and treatment.

Malignant hyperthermia is genetically transmitted through an autosomal dominant inheritance pattern with variable penetrance. In obtaining a medical history, it is important to document family history of adverse outcomes to general anesthesia. If it is reported that a first-degree relative has had a malignant hyperthermia crisis or susceptibility, then the patient should not be exposed to triggering agents. Anesthetic agents that trigger malignant hyperthermia include: halothane, enflurane, isoflurane, desflurane, sevoflurane, and succinylcholine. Nitric oxide can be used if the anesthesia machine is “vapor-free” and contains no traces of volatile gas. Other safe agents include nondepolarizing muscle relaxants (such as vecuronium, rocuronium, and pancuronium), all ester and amide local anesthetics, ketamine, propofol, etomidate, barbiturates, opiates, and benzodiazepines.

Although it is safe to undergo minor procedures with administration of a topical or local anesthetic agent, patients undergoing complex procedures with intravenous sedation, general anesthesia, or major conduction blockade should be referred to an accredited ambulatory surgical center or hospital. If symptoms are recognized in the operating room (high temperature, increased end-tidal CO2, muscle rigidity), rapid treatment with dantrolene sodium is the highest priority. Acute episodes may require stopping the procedure and transfer to an intensive care unit.

How well did you know this?
1
Not at all
2
3
4
5
Perfectly
15
Q
A 42-year-old woman, with a history of anaphylactic reaction to procaine, comes to the office for consultation regarding augmentation mammaplasty. Anesthetics that contain which of the following should be avoided in this patient?
A) Acetaldehyde
B) Epinephrine
C) Iodine
D) Methylparaben
E) Para-aminobenzoic acid
A

The correct response is Option E.

True allergic reactions to local anesthetics are rare. Type I hypersensitivity reactions may include anaphylaxis, and are modulated by immunoglobulin E. Both ester-based local anesthetics, such as procaine, and amide-based local anesthetics may induce an allergic response. The most likely allergen is para-aminobenzoic acid (PABA). This compound consists of a benzene ring substituted with an amino group at the 4-position (leading to the analogous label of 4-aminobenzoic acid) as well as a carboxyl group. Preservatives such as methylparaben have also been shown to cause allergic reactions to local anesthetics, but are less likely than allergic reactions caused by PABA.

Epinephrine can induce vasoconstriction and increase the safe dosage of administered local anesthetic, though it would not be expected to cause an allergic response itself. Iodine and acetaldehyde should not be found in local anesthetic mixtures.

How well did you know this?
1
Not at all
2
3
4
5
Perfectly
16
Q
An otherwise healthy 52-year-old woman with a family history of cardiac disease undergoes suction-assisted lipectomy of the flanks, thighs, and abdomen using a tumescent technique. She returns to the emergency department 6 hours after discharge because of slurred speech and restlessness. Which of the following is the most likely diagnosis?
A) Fat embolism
B) Lidocaine toxicity
C) Parietal stroke
D) Pulmonary embolism
E) Third spacing
A

The correct response is Option B.

Because lidocaine absorbs slowly from fat, infiltrate solutions that contain up to 35 mg/kg of lidocaine are generally considered safe. Nonetheless, lidocaine toxicity is still a risk of the procedure. In tumescent solution with epinephrine, peak plasma lidocaine levels occur approximately 10 to 14 hours after infiltration, and thus, the presentation 6 hours after discharge is consistent with peak plasma concentration.

Lidocaine toxicity has symptoms of neurologic or cardiac toxicity. In the early stages, the complications are primarily neurologic and can include slurred speech, restlessness, tinnitus, and a metallic taste, as well as numbness of the mouth. As the concentrations increase, the neurologic concentrations become more severe, and can progress to muscle twitching, seizures, and cardiac arrest. Treatment of lidocaine toxicity is supportive.

Fat embolism presents as a petechial rash, respiratory dysfunction, and cerebral dysfunction, and the symptoms usually appear 24 to 48 hours after surgery. Pulmonary embolism presents as leg pain and edema, tachycardia, and low-grade fevers.

Parietal strokes usually cause sensory symptoms, self-perception anomalies, and left-right agnosia. Third spacing refers to fluid shifts into interstitial spaces and can cause edema, hypotension, and decreased cardiac output.

How well did you know this?
1
Not at all
2
3
4
5
Perfectly
17
Q
A 40-year-old woman with a history of severe postoperative nausea and vomiting is scheduled for exchange of bilateral breast tissue expanders for permanent silicone implants. Use of which of the following medications is most likely to decrease the chance of postoperative nausea?
A) Bupivacaine
B) Fentanyl
C) Isoflurane
D) Midazolam
E) Nitrous oxide
A

The correct response is Option A.

Addition of local anesthetics during general anesthesia, whether by subcutaneous, tumescent, or regional block infiltration, can result in decreased dosage requirements of the common sedatives and analgesics that can result in nausea and emesis.

Common anesthetic agents that promote nausea and emesis include opioids (fentanyl, hydromorphone, morphine) and inhalationals (halothane, isoflurane, nitrous oxide). Propofol is currently the most commonly used intravenous agent. It does not appear to directly result in nausea, but it has limited analgesic effects. Therefore, effective anesthesia with propofol requires addition of opioid narcotics (which cause nausea) and/or local anesthetics such as lidocaine and bupivacaine (which may decrease the narcotic requirement).

Midazolam is a sedative-hypnotic that has anxiolytic and amnesic effects, both of which are helpful adjuncts to the surgical patient experience. Nausea is possible with midazolam, but less commonly reported than with narcotic and inhalational agents.

The cause of postoperative nausea and vomiting is multifactorial and not fully understood. Strategies for prevention include:

Recognition of high-risk patients (females, nonsmokers, history of motion sickness, previous postoperative nausea, general anesthesia)
Pre- and postoperative treatment with multiple modalities
(such as scopolamine, ondansetron, aprepitant, corticosteroids) Supplemental intraoperative oxygen
and hydration

How well did you know this?
1
Not at all
2
3
4
5
Perfectly
18
Q
A 54-year-old woman has onset of ventricular fibrillation and severe hypotension 5 minutes after 30 mL bupivacaine 0.5% is administered to the ankle for postoperative pain control during reconstruction of the foot. After initiation of cardiopulmonary resuscitation, intravenous administration of which of the following is the most appropriate management?
A) Atropine
B) Dantrolene
C) Flumazenil
D) Lipid emulsion
E) Metoprolol
A

The correct response is Option D.

The most appropriate management of acute bupivacaine toxicity is a bolus and infusion of 20% lipid emulsion. Every facility where local anesthetic is used in large doses should have a lipid rescue kit clearly labeled and available should the need arise. Although lipid rescue mechanism of action is not completely understood, it may be that the added lipid in the bloodstream acts as a “sink,” allowing for the removal of lipophilic toxins from affected tissues. Major local anesthetic toxicity can have such symptoms as sudden loss of consciousness, tonic-clonic seizures, hypertension followed by progressive hypotension, tachycardia, ventricular fibrillation, bradycardia, asystole, and cardiac arrest. Arrhythmias may be refractory to treatment, and resuscitation may be prolonged, sometimes requiring more than 1 hour.

In the event of a local anesthetic toxicity event, airway management, seizure suppression, and, if needed, cardiopulmonary resuscitation should be performed. Alert the nearest facility having cardiopulmonary bypass capability and administer 20% lipid emulsion (values in parentheses are for 70 kg) as follows:

Bolus 1.5 mL/kg intravenously over 1 minute (~100 mL)
Continuous infusion 0.25 mL/kg/min (~500 mL over 30 minutes)
Repeat bolus every 5 minutes for persistent cardiovascular collapse
Double infusion rate if blood pressure returns but remains decreased
Continue infusion for a minimum of 30 minutes
Although beta-adrenergic blockers may be useful in treating the excitatory cardiovascular phase of local anesthetic toxicity, the potential to progress to more advanced phases with myocardial depression and collapse preclude their routine use. In addition to lipid emulsion, the treatment for local anesthetic–induced cardiac toxicity is generally supportive, and may include amrinone, closed-chest cardiac massage, and cardiopulmonary bypass.

Flumazenil is used to reverse the effects of benzodiazepine toxicity.

Dantrolene is administered in the acute treatment of malignant hyperthermia.

Atropine and dopamine are administered as part of the Advanced Cardiac Life Support protocol for bradycardia or asystole and would not be used in the scenario described.

How well did you know this?
1
Not at all
2
3
4
5
Perfectly
19
Q
A 16-year-old boy who has asthma is brought to the emergency department 3 hours after accidentally injecting the index finger of the nondominant hand with his epinephrine auto-injector. On examination, the finger is cool, pale, and painful. Which of the following drugs works to competitively antagonize the sympathomimetic effects of epinephrine?
A) Lidocaine
B) Marcaine
C) Nitroglycerin paste
D) Phentolamine
E) Prostacyclin
A

The correct response is Option D.

Epinephrine use in hand surgery is becoming more common as is the inadvertent self-injection by people who carry epinephrine injectors (EpiPens). Typically, there is little treatment needed other than supportive care. However, when concern for tissue viability is raised or there is marked pain, subcutaneous phentolamine is the drug of choice to reverse the sympathomimetic effects of epinephrine. Plain lidocaine (typically 2% or more) will cause vasodilation but by a different mechanism than the reversal of the epinephrine. Topical nitroglycerin paste has been used for reversal of vasospasm, but again, a different mechanism is used.

Marcaine is an amide anesthetic that inhibits sodium ion channels. It is not an antagonist of epinephrine.

How well did you know this?
1
Not at all
2
3
4
5
Perfectly
20
Q
A 53-year-old woman comes to the office for removal of multiple nevi. On injection of lidocaine, which of the following signs and symptoms is most likely to suggest lidocaine toxicity in this patient?
A) Bronchospasm
B) Hypertension
C) Tachycardia
D) Tinnitus
E) Urticaria
A

The correct response is Option D.

Signs and symptoms of lidocaine toxicity include dizziness, agitation, lethargy, tinnitus, metallic taste, perioral paresthesia, slurred speech, euphoria, hypotension, and bradycardia.

Tachycardia is not a sign of lidocaine toxicity. Bradycardia is more common.

Bronchospasm and urticaria are not signs of lidocaine toxicity.

How well did you know this?
1
Not at all
2
3
4
5
Perfectly
21
Q

An otherwise healthy 22-lb (10-kg), 2-year-old boy undergoes extirpation of a 5 × 5-cm arteriovenous malformation of the face. To decrease intraoperative blood loss, infusion of which of the following solutions around the lesion is most appropriate?
A) 20 mL of 0.25% bupivacaine with 1:100,000 epinephrine
B) 20 mL of 0.25% bupivacaine with 1:200,000 epinephrine
C) 20 mL of 1.0% lidocaine with 1:100,000 epinephrine
D) 20 mL of 1.0% lidocaine with 1:200,000 epinephrine
E) 20 mL of 1:200,000 epinephrine

A

The correct response is Option E.

The most appropriate solution to infuse around the lesion to decrease intraoperative blood loss is 20 mL of 1:200,000 epinephrine. Although the maximum dose of subcutaneous epinephrine is unknown in a healthy child, large amounts have been shown to be safe in patients undergoing liposuction. The most conservative estimate for the amount of epinephrine (1:200,000) that can be safely injected in this child is 30 mL (3 mL/kg) every 10 minutes. Because this estimate was used in the past when halothane anesthesia was being administered (halothane lowered the arrhythmogenic threshold to epinephrine), greater volumes could likely be infused because other inhalational anesthetics are now used.

Twenty mL of 1% lidocaine or 0.25% bupivacaine with epinephrine cannot be given to this 22-lb (10-kg) child because it exceeds the maximum dose. The maximum dose of lidocaine with epinephrine that can be administered is 7 mg/kg; because the concentration of 1% lidocaine is 10 mg/mL, only 7 mL of this solution could be injected. The maximum dose of bupivacaine with epinephrine that can be given is 3 mg/kg; because the concentration of 0.25% bupivacaine is 2.5 mg/mL, only 12 mL of this solution could be administered.

How well did you know this?
1
Not at all
2
3
4
5
Perfectly
22
Q

A 7-year-old boy with a history of anaphylactic reaction to bee stings is brought to the office 45 minutes after epinephrine was mistakenly injected to the tip of the index finger from his automatic injection device. Physical examination of the finger shows swelling, tenderness, and decreased capillary refill. Which of the following is the most appropriate next step in management?
A) Apply a cold compress
B) Apply a warm compress
C) Elevate the extremity
D) Perform a stab incision and saline irrigation
E) Subcutaneously administer an antidote

A

The correct response is Option C.

The most appropriate next step in management is elevation of the extremity. Elevation facilitates venous return of the infiltrate, decreases swelling, and minimizes the risk of skin necrosis. The vasoconstrictive effects of epinephrine only last for 60 to 90 minutes; much longer ischemia times are necessary to cause skin necrosis. For example, amputated digits may be successfully replanted after 33 hours of warm ischemia time.

The use of warm or cold compresses on an infiltration site is controversial. Heat may theoretically stimulate the evacuation of the infiltrate through vasodilation and increased blood flow. Ice may theoretically limit the inflammatory reaction as well as the diffusion of the infiltrated substance by causing vasoconstriction. However, warm or cold compresses may worsen tissue damage. Heat can cause thermal injury, and ice can cause ischemia from vasoconstriction, resulting in a “second hit” at the extravasation site. A partial-thickness skin injury may be converted to a full-thickness wound.

Although phentolamine, an alpha-adrenergic antagonist, has been used to treat epinephrine infiltrations, the added volume of phentolamine can worsen the injury. Injection of an antidote adds more fluid to the subcutis and may increase the risk of pressure necrosis. In addition, because epinephrine causes vasoconstriction for only 60 to 90 minutes, its effects have usually worn off by the time the extravasation is noted, the patient is evaluated by a physician, the phentolamine is ordered, and the antidote is obtained from the pharmacy.

Saline flush-out, using stab incisions through which saline is flushed, may be helpful for the infiltration of chemotherapeutic agents, but it requires several incisions and would be considered after elevation of the extremity.

23
Q

A 48-year-old woman is evaluated because of a unilateral fixed, dilated, and nonresponsive pupil 1 day after she underwent prophylactic mastectomy and immediate reconstruction with a superior gluteal artery perforator free flap. History includes marked postoperative nausea. Multimodal antiemetic prophylaxis therapy was implemented during the procedure. Which of the following is the most likely causal agent of the fixed and dilated pupil?

A) Intravenous dexamethasone
B) Intravenous droperidol
C) Intravenous ondansetron
D) Oral metoclopramide
E) Scopolamine patch
A

The correct response is Option E.

Postoperative nausea and vomiting (PONV) is a distressing complication of general anesthesia and occurs in 25 to 30% of surgeries. Oftentimes, multimodality treatment is implemented for prophylaxis.

Transdermal scopolamine is a potential long-acting prophylactic antiemetic initially developed to prevent motion sickness but approved in 2001 by the US Food and Drug Administration (FDA) for the prevention of PONV. Scopolamine is a centrally acting anticholinergic agent and is designed as a patch placed behind the ear that will deliver 1.5 mg of scopolamine transdermally at a constant rate over 3 days. It has been shown to be effective in decreasing PONV for up to 24 hours after surgery. Several adverse events can occur including sedation, dry mouth, and blurred vision. In addition, mydriasis, usually ipsilateral to the side of patch application, can occur if direct contamination to the eye occurs from rubbing the eyes after manipulating the patch without hand washing. The diagnosis of scopolamine contamination can be confirmed by placing 0.5 to 1.0% pilocarpine hydrochloride in the affected eye. A dilated pupil from pharmacologic mydriasis will not constrict, whereas a dilated pupil from paralytic mydriasis will constrict. Misdiagnosis can result in an unnecessary and extensive workup for an intracranial etiology.

The other options including dexamethasone, droperidol, metoclopramide, and ondansetron are effective antiemetic agents but have no anticholinergic profile.

24
Q
A 23-year-old woman with no history of surgery undergoes augmentation mammaplasty with administration of nitrous oxide, isoflurane, and propofol. Sixty minutes into the procedure, the nurse anesthetist notifies the surgeon that the patient has increasing end-tidal carbon dioxide concentrations, tachycardia, and severe masseter muscle rigidity. Which of the following is the most appropriate next step in management?
A) Administer intravenous saline
B) Administer a muscle relaxant
C) Stop isoflurane
D) Stop nitrous oxide
E) Stop propofol
A

The correct response is Option C.

The patient is experiencing signs of early malignant hyperthermia, which is a life-threatening crisis and requires immediate attention. The typical symptoms of malignant hyperthermia are caused by a hypercatabolic state with increased heart rate, increased breathing rate, increased carbon dioxide production, increased oxygen consumption, acidosis, masseter muscle rigidity, and rhabdomyolysis. Very high temperature (110.0°F [43.0°C]) usually presents late. Even if treated properly, death may occur as brain damage, muscle damage, renal failure, and multiple organ failure ensue.

The malignant hyperthermia crisis is a biochemical chain reaction response that is “triggered” by commonly used general anesthetics and the paralyzing agent succinylcholine within the skeletal muscles of susceptible individuals. Volatile gaseous inhalation anesthetics like sevoflurane, desflurane, isoflurane, and halothane can trigger malignant hyperthermia. The exact incidence of malignant hyperthermia is not known. Estimates vary from a frequency of one in 5000 to one in 65,000. Over 80 genetic defects have been associated with malignant hyperthermia. Malignant hyperthermia susceptibility is inherited with an autosomal dominant inheritance pattern. Children and siblings of a patient with malignant hyperthermia susceptibility usually have a 50% chance of inheriting a gene defect for malignant hyperthermia.

The medical antidote is dantrolene. Additional methods include use of a hypothermia blanket (under/over the patient) and cold isotonic saline for intravenous solution.

25
Q
A 53-year-old man is brought to the emergency department after sustaining a laceration of the index flexor digitorum profundus, superficialis, and radial digital nerve of the nondominant left hand. He is scheduled to undergo urgent repair with single-cuff Bier block anesthesia. Which of the following would be the best reason to use an axillary block in this patient?
A) Age
B) Gender
C) Occupation
D) Surgical duration
E) Urgency of surgery
A

The correct response is Option D.

The advantages of Bier block include reliability with low incidence of block failure, safety with rapid onset and recovery. The block is limited to tourniquet pain often occurring after 20 to 30 minutes and limits its use to shorter procedures on the upper extremities.

Sudden cardiovascular collapse or seizures may occur if local anesthetic is released into the circulation too early. Disease processes in which a tourniquet is contraindicated include Raynaud disease, sickle cell disease, and severe hypertension. Uncooperative patients and young children are also contraindications.

Short duration procedures, including carpal tunnel release, tendon contracture release, foreign body extraction, and trigger finger release, are examples of procedures where Bier blocks may be considered.

Age, gender, occupation, and urgency of surgery are not contraindications to this procedure. Bier block anesthesia is a contraindication in the very young and very old, but not in a 53-year-old patient.

26
Q
A previously healthy 38-year-old woman has onset of a brief tonic-clonic seizure 30 minutes after a lidocaine-based tumescent anesthesia is administered during large-volume liposuction of the abdomen, hips, and thighs. After 3 minutes, she has onset of asystole, and cardiopulmonary resuscitation is initiated. She is unresponsive to the standard ACLS resuscitation protocols for asystole. Administration of which of the following is the most appropriate next step?
A) Dantrolene
B) Dimercaprol
C) Lipid emulsion
D) N-acetylcysteine
E) Naloxone
A

The correct response is Option C.

This is a case of inadvertent lidocaine toxicity with subsequent seizure and cardiac arrest. Furthermore, lipid emulsion has been used with apparent success early in the spectrum of local anesthetic systemic toxicity to preempt cardiac arrest. The role of lipid emulsion has expanded to treatment of cardiac toxicity due to other lipophilic drugs.

Dantrolene is a treatment for malignant hypothermia. Dimercaprol is a chelating agent used for the treatment of heavy metal toxicities. N-acetylcysteine is used as a mucolytic and also in cases of acetaminophen overdose. Naloxone is used to treat narcotic overdose. There are no data to suggest that any of these medications are otherwise helpful in lidocaine toxicity.

27
Q

A 5-year-old girl has subcutaneous extravasation of an epinephrine infusion in the upper extremity. On examination 1 hour later, the extremity is swollen, nontender, and well perfused. Which of the following is the most appropriate next step in management?
A)Application of a cold compress
B)Application of a warm compress
C)Elevation of the extremity
D)Incision and saline irrigation
E)Subcutaneous administration of phentolamine

A

The correct response is Option C.

The most appropriate next step in management is elevation of the extremity. Elevation facilitates venous return of the extravasate, reduces swelling, and minimizes the risk of skin necrosis.

The use of warm or cold compresses on the extravasation site is controversial. Heat may theoretically stimulate the evacuation of the infiltrate through vasodilation and increased blood flow. Ice may theoretically limit the inflammatory reaction, as well as the diffusion of the infiltrated substance, by causing vasoconstriction. However, warm or cold compresses may also worsen tissue damage. Heat can cause thermal injury, and ice can cause ischemia from vasoconstriction resulting in a ?second hit? at the extravasation site; a partial-thickness skin injury may be converted to a full-thickness wound.

Although phentolamine, an alpha-adrenergic antagonist, has been used to treat epinephrine extravasation, the added volume of phentolamine can worsen the injury. Injection of an antidote adds more fluid to the subcutis and may increase the risk of pressure necrosis. In addition, because epinephrine causes vasoconstriction for only 60 to 90 minutes, its effects have usually worn off by the time the extravasation is noted, the patient is evaluated by a physician, the phentolamine is ordered, and the antidote is obtained from the pharmacy.

Saline flush out, or using stab incisions through which saline is flushed, may be helpful for the extravasation of chemotherapeutic agents, but it requires several incisions and would be considered only after elevation of the extremity.

28
Q

An otherwise healthy 17-year-old boy is brought to the emergency department after sustaining a laceration of the distal middle toe. On examination, the toe is bleeding. Infusion of which of the following local anesthetics is most appropriate at the wound site before repair?
A) 0.5% Lidocaine
B) 1% Lidocaine
C) 1% Lidocaine with sodium bicarbonate
D) 1% Lidocaine with 1:200,000 epinephrine
E) 2% Lidocaine

A

The correct response is Option D.

The most appropriate local anesthetic to infuse at the wound site before repair is 1% lidocaine with 1:200,000 epinephrine. The addition of epinephrine to the local anesthetic increases the safety of the lidocaine and facilitates the laceration repair. Epinephrine slows the absorption of the local anesthetic, which allows lower doses to be used. Epinephrine also increases the duration of action of lidocaine, which provides longer pain relief. Vasoconstriction from the epinephrine reduces bleeding at the site of the injury, which shortens the operative time, lowers the risk of iatrogenic injury, and facilitates the repair.

Epinephrine is not contraindicated in the fingers or toes. The vasoconstrictive effects of epinephrine only last for 60 to 90 minutes; much longer ischemia times are necessary to cause skin necrosis. For example, amputated digits may be successfully replanted after 33 hours of warm ischemia time.

Although epinephrine diluted to a concentration of 1:1,000,000 (which is commonly used in tumescent solution for liposuction) will cause vasoconstriction, its onset is prolonged and its duration of action is shortened compared with epinephrine 1:200,000.

Lidocaine or bupivacaine without epinephrine does not reduce bleeding at the site of injury, which increases the difficulty of the repair. In addition, without epinephrine, lower doses of local anesthetic must be used, and the patient has a shorter duration of pain relief.

29
Q
A 5-year-old girl undergoes repair of a forehead laceration in the emergency department. Administration of ketamine is used for sedation. Which of the following best describes intravenous administration of ketamine when compared with intramuscular administration in this patient?
A)Higher rate of laryngospasm
B)Longer duration of effect
C)Longer time to clinical onset
D)Lower rate of vomiting
A

The correct response is Option D.

Ketamine can be administered intravenously or intramuscularly. Intravenous injection is generally safer, and has a lower rate of laryngospasm, shorter duration of clinical onset and effect, and lower rate of vomiting. It can be titrated with a continuous infusion or repeated boluses to effect.

30
Q
Which of the following medications is most likely to trigger malignant hyperthermia?
A) Dantrolene
B) Isoflurane
C) Nitrous oxide
D) Pancuronium
E) Propofol
A

The correct response is Option B.

Malignant hyperthermia is a rare but potentially fatal sensitivity to volatile anesthetics and depolarizing neuromuscular blocking agents. A commonly used volatile inhalation agent is isoflurane. Succinylcholine is a depolarizing muscle relaxant frequently present in the operating room. When genetically susceptible individuals are given anesthesia with these agents, the classic findings include skeletal muscle rigidity, tachycardia, fever, cardiac arrhythmias, and metabolic and respiratory acidosis, leading to severe hypotension. Hyperthermia, the hallmark of malignant hyperthermia, is most often a late sign. The earliest signs are tachycardia and an increase in the end-expired carbon dioxide concentration.

Dantrolene is not a trigger for malignant hyperthermia. It is the only drug recommended for the treatment of malignant hyperthermia.

Propofol and nitrous oxide are not malignant hyperthermia triggers. Pancuronium is a non-depolarizing muscle relaxant and is therefore safe to use.

31
Q
A 23-year-old woman is scheduled to undergo augmentation mammaplasty with intravenous administration of a sedative and local injection of 1% lidocaine with 1:100,000 epinephrine. Weight is 110 lb (50 kg). Which of the following is the maximum dose of lidocaine with epinephrine that can be administered to this patient?
A) 20 mL
B) 25 mL
C) 30 mL
D) 35 mL
E) 40 mL
A

The correct response is Option D.

The maximum dose of lidocaine with 1:100,000 epinephrine recommended for nontumescent injection is 7 mg/kg. In the 110-lb (50-kg) patient described, the maximum dose is 350 mg. One percent lidocaine with 1:100,000 epinephrine contains 10 mg per 1 mL; therefore, the maximum dosage for injection is 35 mL.

32
Q
An otherwise healthy 45-year-old woman undergoes excisional biopsy of a skin lesion on the chest under local anesthesia with 30 mL of 1% lidocaine with 1:100,000 epinephrine. Weight is 176 lb (80 kg). The patient says she has light-headedness, headache, and palpitations 5 minutes into the case after administration of anesthesia. Cyanosis and tachycardia are noted. Oxygen saturation is 90%, and supplemental oxygen is administered. Methemoglobinemia is suspected. Administration of which of the following antidotes is the most appropriate next step?
A) Dantrolene
B) Glucagon
C) Insulin
D) Methylene blue
E) Propranolol
A

The correct response is Option D.

The most appropriate next step in management is the administration of methylene blue as an antidote for methemoglobinemia. Methemoglobinemia is a clinical condition in which hemoglobin is unable to carry oxygen to end organs due to the formation of methemoglobin. Red blood cells contain four hemoglobin chains. Each hemoglobin molecule is composed of four polypeptide chains associated with four heme groups. The heme group contains an iron molecule in the reduced or ferrous form (Fe2+). By sharing an electron in this form, iron can combine with oxygen to form oxyhemoglobin. Hemoglobin can accept and transport oxygen only when the iron atom is in its ferrous form. When hemoglobin loses an electron and becomes oxidized, it is converted to the ferric state (Fe3+), or methemoglobin. Methemoglobin lacks the electron that is needed to form a bond with oxygen and, thus, is incapable of oxygen transport. Local anesthetics can act as oxidizing agents for the ferrous form of iron. Oxygenated blood is red, deoxygenated blood is blue, and blood containing methemoglobin is a dark, reddish brown color. This dark hue imparts clinical cyanosis when methemoglobin levels are at 1.5 g/dL (approximately 10 to 15% methemoglobin concentration); however, a level of 5 g/dL of deoxygenated blood is required for similar effects. Therefore, when methemoglobin levels are relatively low, cyanosis may be observed without cardiopulmonary symptoms. Normal methemoglobin concentrations are 1% (range of 0 to 3%). At concentrations of 3 to 15%, a slight discoloration (i.e., pale, gray, or blue) of the skin may be present. At concentrations of 15 to 20%, the patient may be relatively asymptomatic, but cyanosis is likely to be present. Signs and symptoms at concentrations of 25 to 50% include headache, dyspnea, light-headedness, weakness, confusion, palpitations, and chest pain. Cardiac arrhythmias, delirium, seizures, profound acidosis, coma, and death can occur at concentrations of 50 to 70%. Administration of supplemental oxygen, removal of the offending oxidizing agent, and the administration of the antidote methylene blue are the mainstays of therapy. Methylene blue acts to reduce the heme group from methemoglobin to hemoglobin. Most patients require only one dose. Resolution of toxicity should be seen within 1 hour, often within 20 minutes. The most appropriate dosage for adults is 1 to 2 mg/kg (0.1 to 0.2 mL/kg) intravenously over 3 to 5 minutes; the dose is repeated in 1 hour if continued symptomatology or significant methemoglobinemia is noted. The total dose should not exceed 7 mg/kg. Although there are reports of successful usage in neonates and infants, administration of methylene blue is not recommended for pediatric patients younger than age 6 years. For patients older than age 6 years, dosage is individualized; most cases reported in medical literature have utilized starting doses of 1 mg/kg either ntravenously, intramuscularly, or intraosseously over a period of 5 minutes.

Most cases of local anesthetic-related methemoglobinemia have been associated with topical benzocaine (1 in 7000 bronchoscopies in one study). However, there are cases of lidocaine associated with this potentially fatal reaction in the literature, and knowledge of this is vitally important given the frequency with which plastic surgeons use this drug.

Glucagon is an antidote for beta-blocker overdose, propranolol is a beta-blocker utilized in cardiac arrhythmias, insulin is used to decrease glucose levels or in scenarios of hyperkalemia, and dantrolene would be used in malignant hyperthermia.

33
Q

A 62-year-old woman is undergoing excision of a ganglion on the dorsal wrist with lidocaine Bier block. During the procedure, she says she feels pain, becomes restless progressively, develops a metallic taste in the mouth, and has ringing in the ears. Which of the following is the most appropriate management?
A) Administer diphenhydramine intravenously and continue with the procedure
B) Decrease the tourniquet pressure by 50 mmHg to ease the tourniquet pain
C) Deflate the tourniquet completely, wait a few minutes to reperfuse the arm, then exsanguinate and reinflate the tourniquet
D) Inject bupivacaine (Marcaine) locally to help with the pain and continue with the procedure
E) Maintain an airway and administer oxygen

A

The correct response is Option E.

The clinical scenario describes tourniquet cuff leak leading to lidocaine toxicity. Factors that predispose to cuff leak during Bier block anesthesia include obesity (funnel-shaped arms) and hypertension. Initial signs of lidocaine toxicity include anxiety, tinnitus, and perioral numbness; muscular twitching, seizures, and respiratory or circulatory arrest may develop later. Critical initial management includes maintenance of the airway, oxygen ventilation, and intravenous fluid administration.

Administering diphenhydramine intravenously and continuing with the procedure is inappropriate. Diphenhydramine should not be administered because the patient’s symptoms are not consistent with an allergic reaction.

Deflating the tourniquet completely, waiting a few minutes to reperfuse the arm, then exsanguinating and reinflating the tourniquet, and deflating the tourniquet by 50 mmHg to ease the tourniquet pain, are both inappropriate. Deflating the tourniquet will worsen the patient’s condition by increasing the serum concentration of lidocaine.

Administering bupivacaine (Marcaine) locally to manage the pain and continuing with the procedure is inappropriate because the underlying problem is not incisional pain. Continuing with the procedure without addressing the patient’s distress may lead to the development of more serious complications.

34
Q

A 64-year-old woman is evaluated because of pulmonary distress that begins in the postanesthesia care unit after undergoing a 3-hour elective abdominoplasty and progressively worsens during the next 6 hours. History includes long-standing hypertension that is well controlled with metoprolol. She smokes one pack of cigarettes per week, but she discontinued smoking 2 weeks before the surgery. Preoperative vital signs were within normal limits. Intraoperative studies show:

Crystalloid administration

4000 mL

Blood loss

250 mL

Urine output

125 mL

Postoperative studies show:

Nonrebreather face

mask flow rate

10 L/min to maintain

oxygen saturation > 90%

Heart rate is 80 bpm, respirations are 24/min, and blood pressure is 162/95 mmHg. Evaluation shows no abdominal discomfort, chest pain, or changes in mental status. A portable chest x-ray study shows pulmonary venous congestion. ECG shows sinus rhythm, signs of left ventricular hypertrophy, and normal S-T segments. Which of the following is the most likely cause of the pulmonary failure?
A ) Acute coronary syndrome

B ) Congestive heart failure

C ) Exacerbation of chronic obstructive pulmonary disease

D ) Pneumonia

E ) Pulmonary embolism

A

The correct response is Option B.

The patient described has developed early postoperative pulmonary failure most likely caused by congestive heart failure. Perioperative fluids were administered with the assumption that the patient had normal cardiac function. Her history of long-standing hypertension, combined with postoperative studies that included the ECG showing left ventricular hypertrophy and chest x-ray study consistent with pulmonary edema, support diastolic dysfunction.

Congestive heart failure is currently the leading admission diagnosis for medicine and cardiology hospital services. The main reason is the overlooked high prevalence of diastolic dysfunction secondary to long-standing systemic hypertension. Patients with hypertension and cardiac diastolic dysfunction have preserved left ventricular contractile function, but they also have hypertrophied ventricular muscle that is unable to maintain normal diastolic compliance. This can lead to higher left ventricular filling pressures, elevated atrial pressures, atrial distension, atrial arrhythmias, elevated postcapillary pulmonary pressures, ventilation/perfusion mismatches, and pulmonary and peripheral edema. Treatment involves afterload and preload reduction.

Acute coronary syndrome is caused by an atherosclerotic plaque rupture blocking a coronary artery. The acute event is usually associated with chest pain and elevation of S-T segments on ECG. Pulmonary embolism is always a concern, but that usually occurs later in the postoperative period. Onset of symptoms is sudden and characterized by chest pain, respiratory distress, and anxiety. Pneumonia and chronic obstructive pulmonary disease exacerbation are not supported by the clinical history.

35
Q

A 24-year-old man who sustained burns over 60% of the total body surface area in a chemical fire requires skin grafting for release of ectropion after 8 weeks in the intensive care unit. When administering anesthesia, which of the following drugs is most likely to have adverse effects in this patient?

A ) Atropine

B ) Diazepam

C ) Enflurane

D ) Pentobarbital

E ) Succinylcholine

A

The correct response is Option E.

Succinylcholine-induced hyperkalemia is most frequently observed in patients with large burns who have also had long hospitalizations. Without treatment, potentially fatal cardiac arrhythmias can occur. This condition is also observed in patients who have endured long hospitalizations for neurologic disorders, limiting mobility. Upregulation of acetylcholine receptors is thought to be responsible for the increased efflux of potassium from muscle cells when depolarizing neuromuscular blocking drugs such as succinylcholine are administered to such patients. Treatment includes supportive measures and administration of calcium, glucose, insulin, albuterol, furosemide, and/or sodium polystyrene sulfonate (Kayexalate), depending on the patient €™s symptoms.

Benzodiazepines are widely used and well tolerated as anxiolytic agents in all types of patients. Burn patients are not at increased risk for side effects, and use of these agents preoperatively for the patient described would be appropriate. Although halothane can induce malignant hyperthermia, especially in combination with succinylcholine, this complication is caused by a genetic predisposition. Although intravenous barbiturates cause severe respiratory depression and drowsiness, prolonging the postoperative recovery, they are nevertheless safe in young patients, including a burn patient. Belladonna alkaloids are used to chemically lower the heart rate (atropine) and prevent nausea (scopolamine), and they are not contraindicated in a young, otherwise healthy burn patient.

36
Q

An otherwise healthy 38-year-old man comes to the emergency department after inadvertently hitting the tip of the index finger of the nondominant left hand with a hammer. He has smoked one pack of cigarettes daily for 20 years. Physical examination shows a subungual hematoma from a nail bed laceration, as well as edema and ecchymosis of the finger tip. There are no exposed vital structures. To repair the injury, a digital block is performed with 1% lidocaine in 1:200,000 epinephrine. Two hours after administration of the local anaesthetic, the finger remains pale and cool. Which of the following is the most appropriate next step in management?
A ) Administration of low-dose aspirin

B ) Application of a warm compress to the affected digit

C ) Elevation of the hand

D ) Injection of phentolamine

E ) Topical application of papaverine to the digit

A

The correct response is Option D.

While it has been traditionally taught that the use of epinephrine-containing local anesthesia in performing digital blocks is to be shunned, recent peer-reviewed literature suggests that this is a potentially safe practice. There have been no reported cases of digital gangrene after commercially available lidocaine with epinephrine was introduced in 1948, except for reports of inadvertent iatrogenic injection with 1:1000 epinephrine. The previous reports of digital ischemic necrosis were all associated with older anesthetics, such as cocaine, eucaine, and procaine, which were mixed with epinephrine by hand. Furthermore, those older reports relate excessive volumes of injection, prolonged use of the tourniquet, use of hot soaks, and concomitant infection.

Regardless of current practice or teaching, however, if an epinephrine-containing compound is to be administered to the digit, the surgeon must be familiar with how to reverse the pharmacologically induced vasoconstriction. In these situations, local infiltration of phentolamine, an alpha-receptor blocker, is able to reverse the effects of epinephrine and salvage the digits.

Ingestion of low-dose aspirin will not counter vasoconstriction. It is an option for use in the replanted digit to help promote vascular patency, although this has not been definitely proven.

Although application of warm compresses is a potential treatment for hypothermia-induced vascular embarrassment, it would not reverse the pharmacologically induced effects of the epinephrine, and therefore is not the most appropriate choice for the patient described.

Elevation of the hand will not reverse the epinephrine-induced vasospasm.

Topical administration of a calcium channel blocker, such as papaverine, would also not have the desired effect of reversing epinephrine-induced vasospasm when applied as described. Topical administration has been used in digital replantation to help reverse vasospasm. However, in the scenario described, the digital vessels are not exposed, and direct injection of a reversal agent is the most appropriate, direct method of addressing this situation.

37
Q

Which of the following anesthesia techniques is contraindicated in a 44-year-old man scheduled to undergo open carpal tunnel release?

A ) General anesthesia

B ) Local anesthesia with lidocaine and epinephrine

C ) Local anesthesia with lidocaine only

D ) Regional anesthesia with a Bier block

E ) Regional anesthesia with a nerve block at the cubital tunnel

A

The correct response is Option E. A nerve block at the cubital tunnel, through which the ulnar nerve passes, will provide appropriate anesthesia for the ulnar nerve, but not for the median nerve, which passes anterior to the cubital tunnel at the elbow and through the carpal tunnel at the wrist. This would not provide sufficient anesthesia to perform carpal tunnel release. Both general anesthesia and regional anesthesia with a Bier block are acceptable choices for use in hand surgery. When used in hand surgery, local anesthesia with epinephrine has been documented to be safe, despite incorrect dogma that epinephrine is dangerous in this anatomical region. Several recent critical analyses demonstrate that epinephrine neither causes significant ischemia in the hand nor risks digital infarction.

38
Q

A 59-year-old woman is scheduled to undergo blepharoplasty. She has a history of postoperative nausea and vomiting. Administration of which of the following anesthetic agents is most appropriate for this patient?

A ) Inhalational nitrous oxide

B ) Inhalational sevoflurane

C ) Intravenous etomidate

D ) Intravenous ketamine

E ) Intravenous propofol

A

The correct response is Option E.

Postoperative nausea and vomiting (PONV) is a multifactorial entity, comprising patient, surgical, and anesthetic factors. Attempts have been made to identify the risk factors. A recent study concluded that female gender, a history of motion sickness or PONV, nonsmoking status, and use of postoperative opioids were most predictive. The incidences of PONV with the presence of zero, one, two, three, or all four of these risk factors were 10%, 21%, 39%, 61%, and 79%, respectively. Some surgical procedures are associated with a higher incidence of PONV, eg, major breast procedures, strabismus surgery, laparoscopy, and laparotomy. Agents used during anesthesia, including opioids, nitrous oxide (N2O), and volatile inhalational anesthetics, are emetogenic. Pain, anxiety, and dehydration may also increase the incidence of PONV.

Propofol is widely used in ambulatory surgery because of its favorable clinical characteristics, including rapid recovery and minimal PONV. When compared with propofol, sevoflurane is associated with a higher incidence of PONV. Etomidate has minimal cardiovascular side effects, but it has a high incidence of PONV and is not commonly used in outpatient anesthesia. Nitrous oxide has been found to increase PONV when used as a primary anesthetic agent but not when used in combination with other agents. Intravenous anesthesia with ketamine has a lower risk of pulmonary aspiration but may result in higher rates of PONV.

39
Q

A 25-year-old man with profound hyperhidrosis is undergoing botulinum toxin type A (BOTOX Cosmetic) injection. EMLA (prilocaine-lidocaine) cream is applied to the injection site. Which of the following best represents the minimum amount of time necessary to achieve anesthesia?

A ) 5 Minutes

B ) 15 Minutes

C ) 30 Minutes

D ) 60 Minutes

E ) 90 Minutes

A

The correct response is Option D.

EMLA (eutectic mixture of local anesthetic) is a highly effective method of inducing topical anesthesia. A eutectic mixture consists of two substances that when mixed have a lower melting point than either substance alone. EMLA uses a combination of lidocaine and prilocaine. It has excellent penetration and produces remarkably little sensitivity reaction.

The efficacy of EMLA varies according to the relative thickness of the skin and its vascularity. The cream is applied fairly thickly (1-2 g of EMLA per 10 cm2 of skin) and a dressing such as Opsite or Tegaderm is used to keep it in place. Gentle regular massage to the area can be applied for at least an hour.

40
Q

A 25-year-old woman is undergoing bilateral augmentation mammaplasty. During the procedure, the patient becomes tachycardic, with a sudden rise in end-tidal carbon dioxide, a decrease in oxygen saturation, and elevated temperature. Which of the following is the most appropriate sequence of management?

A ) Administer a beta blocker, volume resuscitate, and finish surgery

B ) Administer therapeutic dose of heparin intravenously and start a heparin drip; rush patient to angiography suite

C ) Open the incision, remove both prostheses, and administer high-dose antibiotics

D ) Place bilateral chest tubes and order a chest radiograph

E ) Stop inhalational agents, hyperventilate with 100% oxygen, and administer dantrolene

A

The correct response is Option E.

Malignant hyperthermia is an inherited myopathy characterized by a hypermetabolic state when the patient is exposed to an appropriate triggering agent, such as all inhalational agents or depolarizing muscle relaxants (ie, succinylcholine). Initial presentation is usually an unexplained tachycardia, followed by an increase in end-tidal carbon dioxide, decrease in oxygen saturation, dysrhythmia, and muscle rigidity. Hyperthermia occurs but can be a late finding.

The most appropriate management involves calling for assistance, stopping all triggering agents, hyperventilating with 100% oxygen, administering dantrolene, stopping all surgery, volume resuscitating, and correcting hyperkalemia.Removal of prostheses and administration of antibiotics are not appropriate because this is not a toxic shock syndrome. Bilateral pneumothoraces are unlikely, although they should be ruled out. Pulmonary embolism is unlikely in the young patient described during a fairly short operation (very low risk).

41
Q
A 44-year-old man is scheduled to undergo extensive body contouring surgery following massive weight loss. Which of the following best represents the most appropriate minimum ambient operating room temperature?
A ) 60 °F (15.6 °C)
B ) 65 °F (18.3 °C)
C ) 70 °F (21.1 °C)
D ) 75 °F (23.9 °C)
E ) 80 °F (26.7 °C)
A

The correct response is Option C.

Long operative times and significant surface area exposure are often required in body contouring procedures following massive weight loss. These factors may contribute to decreases in core body temperature. Actively maintaining intraoperative normothermia allows patients to maintain normal coagulation function during elective plastic surgery lasting longer than two hours, potentially reducing the occurrence of bleeding-related complications.It has been recommended that ambient room temperatures for these cases be kept no less than 70 °F (21.1 °C). Additionally, as much of the body as possible should be covered to reduce heat loss. Additional measures to maintain normothermia include fluid-filled warming blankets placed under the patient on the operating table, warming of intravenous fluids, head and foot stockings, and forced-air warming blankets.

42
Q

The American Association for Accreditation of Ambulatory Surgery Facilities (AAAASF) standards mandate that all facilities must have which of the following drugs readily available for the treatment of malignant hyperthermia?

A ) Dalteparin

B ) Dantrolene

C ) Darifenacin

D ) Dicyclomine

E ) Dobutamine

A

The correct response is Option B.

Dantrolene is the only clinically available drug for the specific treatment of malignant hyperthermia.

Malignant hyperthermia is a rare but potentially fatal sensitivity to volatile anesthetics and depolarizing neuromuscular blocking agents. When genetically susceptible individuals are given anesthesia with these agents, the classic findings include skeletal muscle rigidity, tachycardia, fever, cardiac arrhythmias, and metabolic and respiratory acidosis, leading to severe hypotension. Hyperthermia, the hallmark of malignant hyperthermia, is most often a late sign. The earliest signs are tachycardia and an increase in the end-expired carbon dioxide concentration.

With the use of dantrolene, the mortality from malignant hyperthermia decreased from 80% in 1960 to less than 10%. Dantrolene is a skeletal muscle relaxant that depresses the intrinsic mechanisms of excitation-contraction coupling. The mainstay of treatment for malignant hyperthermia includes discontinuance of the inciting anesthetic agents, rapid body cooling, and dantrolene. Rapid dantrolene preparation and infusion are critical. Therapy is started by continuous rapid intravenous push beginning at a dose of 1 mg/kg to 2.5 mg/kg and continuing until symptoms subside or the maximum cumulative dose of 10 mg/kg is reached. If symptoms reappear, the regimen may be repeated. Dantrolene is packaged in powdered form as a 20-mg vial which must be reconstituted with 60 mL of sterile water for injection (without a bacteriostatic agent). Acidic solutions such as 5% dextrose and 0.9% sodium chloride should not be used. The effective dose to reverse the crisis will vary in each individual. Pediatric dosing is the same as for adults. Care must be taken during infusion to avoid extravasation because there is a possibility for tissue necrosis because of the high pH.

Malignant hyperthermia may not occur during the first exposure to an anesthetic triggering agent, but it may develop during subsequent operations. Symptoms range from mild to life-threatening hyperthermic crisis. The only screening test for susceptibility is through muscle biopsy. Dantrolene should be administered preoperatively to patients prone to malignant hyperthermia; anesthetic drugs that are known to trigger malignant hyperthermia should be avoided. Medications considered safe for patients susceptible to malignant hyperthermia include both lidocaine and bupivacaine. General anesthesia can be performed with alternative anesthetic regimens including nondepolarizing paralytic agents, nitrous oxide, and opioids (eg, vecuronium, propofol [Diprivan], and fentanyl).

Dobutamine is an inotropic agent that may be used to support circulation in severe malignant hyperthermia, but it is not specific for the disease. Dicyclomine is an anticholinergic used to treat irritable bowel syndrome. Darifenacin is a muscarinic receptor antagonist used to treat overactive bladder. Dalteparin is a low-molecular-weight heparin used as deep vein thrombosis prophylaxis.

43
Q

A 57-year-old man has chest pain in the recovery room after undergoing uneventful septorhinoplasty for repair of a deviated septum and collapsed internal nasal valve. Electrocardiography shows changes consistent with acute myocardial ischemia. He has no history of cardiac disease and takes no medications. Weight is 154 lb (70 kg). Intraoperative anesthesia included 1% lidocaine 20 ml with 1:100,000 epinephrine infiltrated into the nasal tissues, followed by intranasal placement of pledgets soaked in 4 ml of 10% cocaine solution. Infraorbital nerve blocks were performed bilaterally using a total of 10 ml of 0.5% bupivacaine. Which of the following is the most likely mechanism of the myocardial ischemia?

(A) Decreased systolic and diastolic left ventricular function

(B) Histamine-induced coronary spasm

(C) Impaired depolarization of the cardiac action potential

(D) Increased peripheral vascular resistance

(E) Vasoconstriction of epicardial coronary arteries

A

The correct response is Option E.

The most likely cause of change in electrocardiography in the patient described is vasoconstriction of the epicardial coronary arteries caused by a toxic dose of cocaine. The maximum safe dose of cocaine is 1.5 mg/kg, or 100 mg in a 70-kg adult. The patient described received a topical dose of 400 mg. With an average 37% mucosal absorption rate, the absorbed dose is still approximately 150 mg. Cocaine increases myocardial oxygen demands via alpha €‘adrenergic stimulation and decreases myocardial oxygen supply through vasoconstriction of the coronary arteries, resulting in myocardial ischemia.

Adverse cardiovascular reactions are possible after the administration of local anesthetics. At toxic levels, lidocaine and bupivacaine block sodium channels in cardiac tissue, resulting in a depressed rate of depolarization during phase 0 of the cardiac action potential. This manifests as prolonged QRS width, depression of left ventricular function, and ventricular arrhythmias from increased activity in reentrant pathways.

Hypersensitivity reactions to local anesthetics usually result in a shock-type scenario with hypotension, although histamine can cause coronary artery spasm. High doses of epinephrine can cause hypertension through increased peripheral vascular resistance. The doses of lidocaine, epinephrine, and bupivacaine that were administered to the patient described were within the range of maximum recommended doses.

44
Q

A 32-year-old man comes to the emergency department because he has a 2-cm laceration of the forehead. He developed urticaria after receiving a local anesthetic during suturing of a laceration of the leg two years ago. Which of the following local anesthetics is most likely to cause hypersensitivity in this patient?

(A) Bupivacaine

(B) Etidocaine

(C) Lidocaine

(D) Mepivacaine

(E) Procaine

A

The correct answer is Option E.

Actual hypersensitivity is rare and accounts for less than 1% of all reactions to local anesthetics. Allergic reactions may be attributed to other factors such as acute toxicity (eg, high plasma levels caused by inadvertent intramuscular injection), psychomotor reactions (eg, patient anxiety or apprehension), pharmacologic properties of local anesthetics, concurrent drug therapy (eg, epinephrine-induced tachycardia), or preservatives such as paraben or sulfites, which may be present in multidose vials.

Local anesthetics with a para €‘aminobenzoic acid (PABA) ester €‘type structure seem to cause most anesthesia €‘related allergic reactions. Documented cross €‘sensitivity has been exhibited within the ester €‘based family of local anesthetics and structurally related compounds (eg, paraben preservatives). PABA is structurally similar to methylparaben. Amide local anesthetics do not metabolize to PABA; therefore, hypersensitivity to amide local anesthetics is rare. Because of possible hypersensitivity, many manufacturers have reformulated some of their products to eliminate methylparaben. Some of these products include Nesacaine-MPF (methylparaben-free), Xylocaine MPF, Polocaine MPF, and Sensorcaine MPF. A ruling by the United States Food and Drug Administration mandated the removal of methylparaben from all local anesthetic dental cartridges. Antioxidants (sodium bisulfite, metabisulfite) are added to local anesthetic products that contain vasoconstrictors (epinephrine, levonordefrin) to prevent biodegradation by oxygen.

45
Q

A 50-year-old woman comes to the office for rhytidectomy. Height is 5 ft 1 in and weight is 110 lb (50 kg). Diazepam 10 mg and oxycodone 5 mg is administered with a sip of water approximately 30 minutes before the procedure is scheduled to begin. Forty-five minutes after 50 ml of 0.5% bupivacaine and 10 ml of 0.5% lidocaine with 1:100,000 epinephrine is injected into the patient €™s face, she becomes confused and disoriented. The ECG monitor shows heart block and wide-complex arrhythmia. Which of the following is the most likely cause of the ECG reading?

(A) Benzodiazepine toxicity

(B) Bupivacaine toxicity

(C) Lidocaine toxicity

(D) Opioid toxicity

(E) Primary undiscovered coronary artery disease

A

The correct response is Option B.

In the scenario described, a medication error has apparently occurred. A staff member has probably misread the surgeon €™s formula for local anesthetic and used bupivacaine in place of lidocaine. Surgeons should be encouraged to prepare their own solutions and read all labels on the medications they use, but in practice, nurses often prepare wetting solutions and injection solutions, and, even in the best of facilities, errors may occur. The accepted maximum dose of bupivacaine is 2.5 mg/kg, and thus anything over 150 mg is considered toxic in a 50-kg patient. The patient described was given 50 ml of a 0.5% solution, which contains 5 mg/ml of medication. Therefore, she was given 250 mg of the medication, well into the toxic range. On the other hand, had she been given lidocaine 0.5% with epinephrine, the maximum dosage would have been 7 mg/kg, or 350 mg of lidocaine. The amount of lidocaine in 50 ml of a 0.5% solution is 250 mg, within the safe zone for this patient. If the solution had been mixed as intended, the patient described would have received 250 mg of lidocaine and 25 mg of bupivacaine, and the resulting total amount would have been within the safe zone and well below the toxic range.

Large doses of bupivacaine can cause irreversible, nonrecoverable heart block (a recent case in Australia involved placing the patient on cardiac bypass until the toxicity was resolved [Soltesz, et al. 2003]). Recent literature has indicated that intralipid (1 ml/kg of a 20% solution IV recommended by Weinberg) is important in reducing mortality (Weinberg, et al. 1998, 2003, 2006; Rosenblatt, et al. 2006). Discussion at recent anesthesiology meetings has resulted in the recommendation that all facilities using bupivacaine also be stocked with intralipid (Dr. P. Schneider, anesthesiologist, personal communication). It is essential to realize that transfer to a hospital center for further treatment is urgent and mandatory in bupivacaine overdose and that immediate transfer without delay offers the best chance of survival, as further absorption from injected tissues is to be expected during this critical period (Mather, et al. 2005).

The dosages of benzodiazepine, lidocaine, and opiates are all appropriate in the scenario described. The ECG changes are consistent with bupivacaine toxicity, not heart ischemia.

46
Q

A 55-year-old woman is scheduled to undergo concurrent suction lipectomy of the neck and lower abdomen and rhytidectomy. Weight is 110 lb (50 kg). Conscious sedation with a local anesthetic and tumescent solution is planned. The neck and face should be injected and this portion of the surgery performed prior to the injection and liposuction of the abdomen for which of the following reasons?

(A) The absorption rate of local anesthetic above the clavicles is two times faster than below the clavicles

(B) The epinephrine must be injected immediately before suctioning or it will not be effective

(C) The facial area requires more anesthesia so the sedation can be lightened during the abdominal portion of the case

(D) The lower abdominal liposuction should be performed first to allow time for the epinephrine to work for the rhytidectomy

(E) The peak absorption of tumescent solution from the abdomen is two hours, and surgery must be completed by this time

A

The correct response is Option A.

One of the key principles of this operation is safety in the amount of local anesthesia used and the timing of injection. The absorption of lidocaine from above the clavicles peaks at approximately five to six hours when injected into the neck via tumescent solution. The absorption of the tumescent peaks at approximately 12 hours for the trunk region (the thighs in the reference below). If the tumescent is given for the abdomen and several hours later for the face and neck, then the lidocaine absorption curves could be superimposed and reach a toxic level. An immediate bolus of all the local anesthetics may not be preferred for a similar reason, as the absorption curve might peak. The epinephrine will be needed in the local anesthetic to delay rapid absorption via vasoconstriction and should not be eliminated if larger doses of local anesthesia are needed, although the epinephrine and local dose can be diluted to give a similar vasoconstrictive effect. Local anesthetics are additive in their risk and have a certain degree of cross-reactivity in their doses. Mixing a less-than-toxic dose of bupivacaine plus a less-than-toxic dose of lidocaine can add to a toxic effect. If large procedures are to be considered under local anesthesia and toxic doses limits are being approached, the surgeon should consider separating operative dates or another choice of anesthesia.

47
Q

A 30-year-old woman at 10 weeks €™ gestation is brought to the emergency department 30 minutes after she sustained injuries to the right hand during a motor vehicle collision. Physical examination shows an open, volarly displaced, and rotated fracture of the fifth metacarpal. Emergency surgery for reduction of the fracture and closure of the wound is planned. Administration of which of the following drugs should be avoided in this patient?

(A) Bupivacaine
(B) Chloroprocaine
(C) Diazepam
(D) Fentanyl
(E) Morphine
A

The correct response is Option C.

The U.S. Food & Drug Administration (FDA) has classified drugs according to their risk during pregnancy. The classifications are as follows:

Category A: Controlled studies in women fail to demonstrate a risk to the fetus in the first trimester (and there is no evidence of a risk in later trimesters). The possibility of harm to the fetus seems remote.

Category B: Either animal-reproduction studies have not demonstrated a risk to the fetus but there are no controlled studies in pregnant women, or animal-reproduction studies have shown an adverse effect (other than a decrease in fertility) that was not confirmed in controlled studies in women in the first trimester (and there is no evidence of a risk in later trimesters).

Category C: Either studies in animals have shown adverse effects on the fetus (teratogenic or embryocidal or other) and there are no controlled studies in women, or studies in women and animals are not available. Drugs should be given only if the potential benefit justifies the potential risk to the fetus.

Category D: There is positive evidence of risk to the fetus in humans, but the benefits from use in pregnant women may be acceptable despite the risk (e.g., if the drug is needed in a life-threatening situation or for a serious disease for which safer drugs cannot be used or are ineffective).

Category X: Studies in animals or humans have demonstrated fetal abnormalities or there is evidence of risk to the fetus based on human experience, or both, and the risk of the use of the drug in pregnant women clearly outweighs any possible benefit. The drug is contraindicated in women who are or may become pregnant.

There is a some evidence that diazepam and other benzodiazepines are teratogenic in humans, increasing the risk of congenital malformations when ingested by the mother during the first trimester of pregnancy. Diazepam is in the FDA pregnancy category D; the other drugs listed are not.

Emergency surgery that requires anesthesia must be taken into consideration because there is a likelihood of initiating labor due to stress and because there is also a risk of drug-induced fetal defects, especially during the first trimester. The anesthetic technique of choice is regional anesthesia. Chloroprocaine is hydrolyzed quickly and, therefore, becomes unavailable for transfer across the placenta. Bupivacaine is strongly bound to protein and, therefore, very little drug is available for transfer across the placenta. If systemic drugs need to be given, morphine and fentanyl are preferred.

48
Q

A 46-year-old woman undergoes suction lipectomy of the abdomen using tumescent anesthesia containing lidocaine. Despite doses of lidocaine higher than recommended for local infiltration, toxicity to the drug does not develop in this patient. Which of the following is the most likely explanation?

(A) Chemical structure of lidocaine
(B) pH of the tissue
(C) Rate of lidocaine absorption
(D) Removal of lidocaine by suction
(E) Toxic threshold for plasma lidocaine concentration
A

The correct response is Option C.

Lidocaine for local infiltration is available in 0.5%, 1%, 1.5%, and 2% solutions that contain epinephrine at a concentration of 1:100,000 or 1:200,000. In contrast, lidocaine for tumescent anesthesia commonly is available in a 0.05% solution that contains epinephrine at a concentration of 1:1,000,000. This dilute solution results in a very slow rate of lidocaine absorption from subcutaneous tissue, which prevents high peak plasma concentrations of the drug as well as toxicity. Although a standard 1% solution yields a maximum plasma concentration of lidocaine in 1 hour, a tumescent solution provides a maximal plasma concentration in 8 to 12 hours.

The chemical structure of lidocaine can vary with the pH of the tissue. However, this is not a primary factor in the pharmacokinetics of tumescent lidocaine. Suction lipectomy removes some lidocaine, which reduces the peak plasma concentration by approximately 25% (compared with a nonBsuction-lipectomy control). However, this is not the primary mechanism that allows a fivefold increase in dose without toxicity. In human studies of the toxic threshold for plasma lidocaine, central nervous system symptoms occurred at concentrations above 4 _g/mL; this was equally true for standard and tumescent solutions.

49
Q

A 27-year-old woman has a true allergy to a local anesthetic agent used in the past. Use of which of the following local anesthetic agents is most appropriate for this patient?

(A) Benzocaine
(B) Chloroprocaine
(C) Lidocaine
(D) Procaine
(E) Tetracaine
A

The correct response is Option C.

True allergic reactions to local anesthetics, although rare, can occur with the ester-linked agents chloroprocaine, tetracaine, procaine, and benzocaine. The degradation product of ester-linked anesthetics is para-aminobenzoic acid (PABA), which is an antigenic substance. The amide-type anesthetics lidocaine, bupivacaine, mepivacaine, and prilocaine do not cause true allergic reactions. Allergic reactions to amide-type anesthetics may occur but are usually related to the preservative methylparaben, which is structurally similar to PABA.

50
Q

Which of the following is LEAST sensitive to increasing plasma levels of lidocaine?

(A) Blood pressure
(B) Central nervous system activity
(C) Heart rate
(D) Muscle tone

A

The correct response is Option A.

Blood pressure is typically insensitive to increasing plasma levels of lidocaine and other local anesthetics because a compensatory increase in systemic vascular resistance prevents the blood pressure from increasing.

Adverse reactions in the central nervous system are much more common and are biphasic. Initially, an excitatory phase occurs, which may be due to inhibition of the amygdala. This phase may produce muscle twitching in the face and extremities followed by tremors that can progress to seizures. As the amount of local anesthetic increases, a depressive phase occurs and is characterized by drowsiness, unconsciousness, and respiratory arrest.

The cardiovascular system is thought to be more resistant than the central nervous system to the effects of local anesthetics. However, it can sustain dangerous reactions, usually at higher plasma levels. With toxic doses of local anesthetics, cardiovascular reactions may include arrhythmias, cardiovascular depression, and shock. Cardiovascular depression tends to be serious and difficult to treat. The more lipid-soluble local anesthetics, such as bupivacaine, tend to have a higher toxicity than the less lipid-soluble drugs, such as lidocaine.

Toxic effects of local anesthetics result from inappropriately high dosage or unintentional intravascular injection. Management of lidocaine toxicity consists of ECG monitoring and oxygen administration. If seizures occur, they are typically controlled with diazepam or midazolam. If mechanical ventilation is required, paralytic agents may be administered. Because of the seriousness of the toxic effects, appropriate monitoring and personnel trained in advanced cardiac life support are required during local anesthetic use, regardless of the magnitude of the procedure.

51
Q

Which of the following best explains the long duration of bupivacaine when compared with lidocaine?

(A) Lipid solubility
(B) Longer onset of action
(C) pKa
(D) Protein binding
(E) Vasoconstrictor activity
A

The correct response is Option D.

The pharmacokinetic properties of local anesthetics, including lipid solubility, pKa, protein binding, and nonspecific tissue binding, determine their clinical characteristics. The duration of action correlates directly with protein binding; ie, greater protein binding results in a longer duration of action. This best explains the long duration of action of bupivacaine when compared with lidocaine; bupivacaine has been shown to be 96% protein-bound, while lidocaine is 64% protein-bound.

The lipid solubility of a local anesthetic determines its potency. Higher solubility leads to a higher potency drug.

Bupivacaine has a longer onset of action (5 to 8 minutes) than lidocaine (2 to 4 minutes), but its duration of action (up to 10 hours) is much longer than that of lidocaine (up to 3 hours). This small difference in onset of action does not account for the prolonged duration of action.

The pKa is defined as the negative logarithm of the acid ionization constant. This determines the onset of action of a local anesthetic because the uncharged (or base) form is the active component. Local anesthetics with a high pKa have the slowest onset of action because the pKa varies more greatly from the physiologic pH of the drug. Lidocaine, with a pKa of 7.9, has a more rapid onset of action than bupivacaine, which has a pKa of 8.1.

Addition of epinephrine, a vasoconstrictor, to a local anesthetic prolongs the duration of action of the anesthetic and decreases its toxicity. However, all local anesthetics, with the exception of cocaine, have natural vasodilator activity.

52
Q

An otherwise healthy 5-year-old child who weighs 22 kg sustains a 3-cm laceration of the right arm in a fall. A 1% lidocaine solution is to be injected prior to suturing. What is the maximum safe dose of lidocaine that should be used in this patient?

(A) 5 mL
(B) 10 mL
(C) 15 mL
(D) 20 mL
(E) 25 mL
A

The correct response is Option B.

In otherwise healthy children older than 3 years of age who have normal lean body mass, the maximum safe dose of lidocaine (Xylocaine) that can be administered is determined by the child’s age and weight. The maximum safe dosage in a 22-kg 5-year-old child is 22 multiplied by 4.5, or 100 mg of lidocaine. At a 1% concentration, which contains 10 mg lidocaine per milliliter (mL), the maximum dose is 10 mL, according to the manufacturer’s recommendation.

53
Q

In a 50-year-old woman who has a history of allergic reaction to tetracaine, which of the following anesthetics should NOT be used?

(A) Bupivacaine
(B) Etidocaine
(C) Lidocaine
(D) Mepivacaine
(E) Procaine
A

The correct response is Option E.

All local anesthetics can be classified as either ester- or amide-type. Ester-type local anesthetics, including procaine, benzocaine, chloroprocaine, and tetracaine, can produce allergic reactions. These anesthetics are metabolized by pseudocholinesterase, forming para-aminobenzoic acid (PABA). This compound triggers antibody formation and lymphocyte stimulation to elicit a hypersensitivity reaction. In one study of patients who had no known history of allergy to local anesthetics, 30% of patients developed a positive skin reaction following intradermal injection of an ester-type anesthetic.

Amide-type local anesthetics, including bupivacaine, etidocaine, lidocaine, and mepivacaine, are used more frequently for local infiltration than ester-type agents. These are stable solutions that do not cause true allergic reactions. Any “allergic” reaction to amide-type local anesthetics is most likely caused by methylparaben, a preservative that is structurally related to PABA. Preservative-free local anesthetics are currently available for use in any patient who has sensitivity to methylparaben. In the same study mentioned above, there were no allergic reactions to the amide-type agents. A trick to help remember which complexes belong to which group is that all amides have an “i” in the prefix before the “caine” (ie, bupivacaine, etidocaine, lidocaine, and mepivacaine).

54
Q

A 5-year-old boy is brought to the emergency department after sustaining a laceration of the left lower extremity. Physical examination shows an isolated 2-cm lesion extending through the dermis. EMLA cream is applied prior to suturing.

The surgeon should wait for how many minutes before suturing the laceration?

(A) 5 minutes
(B) 10 minutes
(C) 20 minutes
(D) 30 minutes
(E) 60 minutes
A

The correct response is Option E.

Because EMLA cream reaches its peak efficacy approximately 45 to 60 minutes after initial application, the surgeon should wait for one hour before suturing the laceration. Studies show that EMLA cream is superior for anesthesia of simple extremity wounds and results in optimal anesthesia in open wounds after approximately 60 minutes. Cumulative sum (CUSUM) analysis has been used to confirm the minimal effective application time.

EMLA cream, a eutectic mixture of local anesthetics, contains lidocaine 5% and prilocaine 5%. Following topical application of EMLA cream, the affected site should be covered with an occlusive dressing.